Gönderen Konu: Diyafont Denklemler Çalışma Soruları ($138$ Tane)  (Okunma sayısı 18149 defa)

Çevrimdışı Metin Can Aydemir

  • G.O Genel Moderator
  • Geo-Maniac
  • ********
  • İleti: 1139
  • Karma: +9/-0
Ynt: Diyafont Denklemler Çalışma Soruları ($138$ Tane)
« Yanıtla #90 : Ekim 24, 2020, 11:04:11 ös »
$84)$ Öncelikle $x$'nin tek sayı olması gerektiğini gösterelim. Aksini varsayalım, $m\in \mathbb{Z}$ olmak üzere, $x=2m$ olsun. Ana denklem, $16m^4+4=py^4$ haline dönüşür. Sol taraf $4$ ile bölündüğünden sağ taraf da bölünmelidir, $p=2$ olsa bile $y$ çift olmak zorundadır. $y=2n$ için $16m^4+4=16pn^4$ olur. Sağ taraf $16$ ile bölünüp, sol taraf bölünmediği için çelişki olur. Dolayısıyla $x$ tek sayıdır. Şimdi ifadeyi çarpanlarına ayıralım, $$x^4+4=x^4+4x^2+4-4x^2=(x^2+2)^2-(2x)^2=(x^2+2x+2)(x^2-2x+2)=py^4$$ bulunur. $x$ tek sayı olduğundan $x^2+2x+2$ ve $x^2-2x+2$ ifadeleri de tek sayıdır. Eğer bu iki ifade aralarında asal değilse, ikisini de bölen bir $q$ tek asal sayısı vardır. $$q\mid x^2+2x+2 \text{         ve        } q\mid x^2-2x+2 \Rightarrow q\mid (x^2+2x+2)-(x^2-2x+2)\Rightarrow q\mid 4x \Rightarrow q\mid x$$ $$q|x \text{         ve        }  q|x^2-2x+2 \Rightarrow q\mid (x^2-2x+2)-x(x-2)\Rightarrow q\mid 2$$ bulunur fakat $q$ tek asal sayı olduğundan bu durum da çelişki oluşturur. Dolayısıyla $x^2+2x+2$ ve $x^2-2x+2$ ifadeleri aralarında asaldır. Bu iki ifade aralarında asal olduğundan ve çarpımları $py^4$ olduğundan $2$ durum söz konusudur.

$i)$ $x^2+2x+2=pa^4$ ve $x^2-2x+2=b^4$ ise $(x-1)^2+1=b^4$ ifadesini düzenleyebiliriz, $$b^4-(x-1)^2=1\Rightarrow (b^2-x+1)(b^2+x-1)=1$$ olur ve buradan gelen tek çözüm $x=1$ durumudur. $x=1$ için ana denklemde $py^4=5$ bulunur, buradan $p=5$ elde edilir.

$ii)$ $x^2+2x+2=a^4$ ve $x^2-2x+2=pb^4$ ise $(x+1)^2+1=a^4$ ifadesini düzenlersek $$(a^2+x+1)(a^2-x-1)=1$$ olur buradan sadece $x=-1$ çözümü gelir. $x=-1$ için ana denklemden yine $py^4=5$ elde edilir. Buradan da $p=5$ gelir.

Tüm durumlarda $p=5$ elde edildiğinden çözüm olmasını sağlayan tek asal sayı $\boxed{p=5}$ 'dir.
Gerçek hikayeler aslında söylenmeyenlerdir.

Çevrimdışı Metin Can Aydemir

  • G.O Genel Moderator
  • Geo-Maniac
  • ********
  • İleti: 1139
  • Karma: +9/-0
Ynt: Diyafont Denklemler Çalışma Soruları ($138$ Tane)
« Yanıtla #91 : Ekim 25, 2020, 02:40:24 öö »
$85)$ Verilen eşitlikleri Aritmetik-Harmonik ortalama eşitsizliğinde kullanırsak, ($k_1,k_2,\dots, k_n$ terimleri pozitif olduğundan bunu uygulayabiliriz.) $$\dfrac{k_1+k_2+\cdots+k_n}{n}\geq \dfrac{n}{\dfrac{1}{k_1}+\dfrac{1}{k_2}+\cdots+\dfrac{1}{k_n}}\Rightarrow \dfrac{5n-4}{n}\geq n\Rightarrow 0\geq n^2-5n+4$$ $n^2-5n+4$ bir parabol olduğundan ve başkatsayısı pozitif olduğundan negatif değer alabilmesi için $n$ sayısını, $x^2-5x+4=0$ denkleminin köklerinin arasında olması gerekir. Bu denklemin çözümleri $x=1$ ve $x=4$ olduğundan $n\in [1,4]$ olmalıdır. $n=1$ ve $n=4$ için yukarıdaki eşitsizlikte eşitlik durumu sağlanacağından terimler birbirine eşit olmalıdır.

$i)$ $n=1$ için $k_1=1$ olur. $(n,k_1)=(1,1)$ çözümü gelir.

$ii)$ $n=4$ için terimlerin eşit olması gerektiğini belirtmiştik. $k_1=k_2=k_3=k_4$ için $k_1+k_2+k_3+k_4=16$ olduğundan $(n,k_1,k_2,k_3,k_4)=(4,4,4,4,4)$ çözümü gelir.

$iii)$ $n=2$ ise $k_1+k_2=6$ ve $\dfrac{1}{k_1}+\dfrac{1}{k_2}=\dfrac{k_1+k_2}{k_1k_2}=\dfrac{6}{k_1k_2}=1$ olduğundan $k_1k_2=6$ elde edilir. Bu iki denklemin tamsayı çözümü yoktur.

$iv)$ $n=3$ ise $k_1+k_2+k_3=11$ ve $\dfrac{1}{k_1}+\dfrac{1}{k_2}+\dfrac{1}{k_3}=1$ denklemleri elde edilir. $$\dfrac{1}{k_1}+\dfrac{1}{k_2}+\dfrac{1}{k_3}=\dfrac{k_1k_2+k_1k_3+k_2k_3}{k_1k_2k_3}=1\Rightarrow k_1k_2+k_1k_3+k_2k_3=k_1k_2k_3$$ olur. $k_3=11-k_1-k_2$ dersek $$k_1k_2+(11-k_1-k_2)(k_1+k_2)=k_1k_2(11-k_1-k_2)$$ eşitliği elde edilir. $k_1k_2=a$ ve $k_1+k_2=b$ için $a+(11-b)b=a(11-b)$ elde edilir. $a$'yı yalnız bırakırsak, $$a+(11-b)b=a(11-b)\Rightarrow a=\dfrac{b^2-11b}{b-10}=b-1-\dfrac{10}{b-10}$$ bulunur. Buradan olası pozitif $b$ değerlerini bulabiliriz, $b=5,8,9,11,12,15,20$ bulunur. Bu değerler için $a$ değerlerini de hesaplayabiliriz. $(a,b)=(6,5),(12,8),(18,9),(0,11),(6,12),(12,15),(18,20)$ ikilileri bulunur fakat bunlardan sadece $(a,b)=(6,5),(12,8),(18,9)$ için $k_1$ ve $k_2$ pozitif tamsayıları bulunabilir. Bu $(a,b)$ ikilileri için $(k_1,k_2,k_3)=(2,3,6)$ ve permütasyonları bulunur.

Tüm çözümler, $(n,k_1,k_2,\dots,k_n)=(1,1),(3,2,3,6),(3,2,6,3),(3,3,2,6),(3,3,6,2),(3,6,2,3),(3,6,3,2),(4,4,4,4,4)$ bulunur.
« Son Düzenleme: Nisan 03, 2024, 08:10:28 ös Gönderen: Metin Can Aydemir »
Gerçek hikayeler aslında söylenmeyenlerdir.

Çevrimdışı Metin Can Aydemir

  • G.O Genel Moderator
  • Geo-Maniac
  • ********
  • İleti: 1139
  • Karma: +9/-0
Ynt: Diyafont Denklemler Çalışma Soruları ($138$ Tane)
« Yanıtla #92 : Ekim 25, 2020, 03:10:42 öö »
$86)$ Öncelikle $x,y,z$ sayılarının farklı olduğu duruma bakalım, genelliği bozmadan $x<y<z$ olsun.

$i)$ $x=1$ için $\dfrac{y+1}{y}\dfrac{z+1}{z}=1$ olur fakat $\dfrac{y+1}{y}>1$ ve $\dfrac{z+1}{z}>1$ olduğundan çözüm yoktur.

$ii)$ $x=2$ ise $\dfrac{y+1}{y}\dfrac{z+1}{z}=\dfrac{4}{3}$ olur. $\dfrac{y+1}{y}>\dfrac{z+1}{z}$ olduğundan $\left (\dfrac{y+1}{y} \right )^2>\dfrac{4}{3}$ olur. Buradan, $y\leq 6$ bulunur. Bu değerler denenirse $(x,y,z)=(2,6,7),(2,5,9),(2,4,15)$ çözümleri bulunur.

$iii)$ $x=3$ ise $\dfrac{y+1}{y}\dfrac{z+1}{z}=\dfrac{3}{2}$ olur. Aynı şekilde $\dfrac{y+1}{y}>\dfrac{z+1}{z}$ dersek $\left (\dfrac{y+1}{y} \right )^2>\dfrac{3}{2}$ olur ve buradan $y\leq 4$ bulunur. $y>x$ olduğundan $y=4$ olmalıdır. yerine yazılırsa $(x,y,z)=(3,4,5)$ çözümü bulunur.

$iv)$ $x>3$ ise $y>4$ ve $z>5$ olur. $\left (1+\dfrac{1}{x}\right )\left (1+\dfrac{1}{y}\right )\left (1+\dfrac{1}{z}\right )<\left (1+\dfrac{1}{3}\right )\left (1+\dfrac{1}{4}\right )\left (1+\dfrac{1}{5}\right )=2$ olur. Çelişki.

Eğer $x,y,z$'den en az ikisi eşitse, genelliği bozmadan $y=z$ olsun. $\left (\dfrac{x+1}{x}\right )\left( \dfrac{y+1}{y}\right )^2=2$ olur. $EBOB(x,x+1)=EBOB(y,y+1)=1$ olduğundan $\dfrac{x+1}{y^2}$ ve $\dfrac{(y+1)^2}{x}$ ifadeleri tamsayı olmalıdır. Bu iki ifadenin çarpımları $2$ olduğundan iki durum söz konusudur.

$a)$ $\dfrac{x+1}{y^2}=1$ ve $\dfrac{(y+1)^2}{x}=2$ ise $y^2=x+1$ ve $y^2+2y+1=2x$ bulunur. İlk denklemdeki eşitliği ikincide yazarsak $x=2y+2$ bulunur. Bunu tekrar ilk denkleme yazarsak $2y+3=y^2$ elde edilir. Bu ikinci derecen denklemi çözersek $y=3$ ve $y=-1$ bulunur. Pozitif tamsayılarda çözdüğümüz için $y=3$ olmalıdır. Buradan $(x,y,z)=(8,3,3)$ çözümü gelir.

$b)$ $\dfrac{x+1}{y^2}=2$ ve $\dfrac{(y+1)^2}{x}=1$ ise $2y^2-1=x$ ve $y^2+2y+1=x$ bulunur. Bu iki denklemden $2y^2-1=y^2+2y+1$ bulunur fakat buradan tamsayı çözüm gelmez.

Tüm çözümler, $\boxed{(x,y,z)=(2,6,7),(2,5,9),(2,4,15),(3,4,5),(8,3,3)}$ ve permütasyonlarıdır.


------------------------------------------------------------------------------------------------------------------------------------
Not: Buradaki gönderide $x,y,z\in \mathbb{Z}^{+}$ için $\left (1+\dfrac{1}{x}\right )\left (1+\dfrac{1}{y}\right )\left (1+\dfrac{1}{z}\right )$ ifadesinin tamsayı olduğu tüm $(x,y,z)$ üçlüleri bulunmuştur.
Gerçek hikayeler aslında söylenmeyenlerdir.

Çevrimdışı Metin Can Aydemir

  • G.O Genel Moderator
  • Geo-Maniac
  • ********
  • İleti: 1139
  • Karma: +9/-0
Ynt: Diyafont Denklemler Çalışma Soruları ($138$ Tane)
« Yanıtla #93 : Ekim 26, 2020, 12:26:08 ös »
$87)$ Genelliği bozmadan $x\geq y\geq z$ olsun. Verilen eşitliği sağlayan $(x,y,z)$ pozitif tamsayı üçlüleri arasından toplamı en küçük olan $(x,y,z)$ alalım. Şimdi verilen eşitliği ikinci dereceden olarak yazalım, $$x^2-x(nyz-2y-2z)+(y+z)^2=0$$ Bu denklemin $x$ dışındaki kökü $a$ olsun. Vietta'dan $x+a=nyz-2y-2z$ ve $xa=(y+z)^2$ bulunur. Bu iki eşitliğinden anlaşılır ki $a$ pozitif bir tamsayıdır. $a\geq x$ olmalıdır çünkü aksi takdirde $(a,y,z)$ bir çözüm olur ve $a+y+z<x+y+z$ olduğundan kabulumuz ile çelişir. Dolayısıyla $a\geq x$'dir. $$(y+z)^2=ax\geq x^2\Rightarrow y+z\geq x$$ olur. Ayrıca $2x\geq y+z$ olduğundan $$(y+z)^2=ax\geq a\left (\dfrac{y+z}{2}\right )\Rightarrow 2(y+z)\geq a$$ olur. Buradan, $$3(y+z)\geq x+a\Rightarrow 3(y+z)\geq nyz-2y-2z\Rightarrow 0\geq nyz-5y-5z\Rightarrow 25\geq n^2yz-5ny-5nz+25=(ny-5)(nz-5)$$ olur. Eğer $n>10$ ise $ny-5> 10y-5>5$ ve benzer şekilde $nz-5>5$ olacağından $(ny-5)(nz-5)>25$ olur ve bu bir çelişkidir. Dolayısıyla $n\leq 10$ olmalıdır. $n=1,2,3,4,5,6,8,9$ için çözüm vardır (çözümün sonunda belirteceğim) fakat $n=10$ ve $n=7$ için çözüm yoktur.

$i)$ $n=10$ ise $25\geq (10y-5)(10z-5)\geq (10-5)(10-5)=25$ olduğundan $y=z=1$ olmalıdır. Ana denklem $(x+2)^2=10x$ haline gelir. $x^2-6x+4=0$ denkleminin tamsayı çözümü yoktur.

$ii)$ $n=7$ ise $y,z\geq 2$ ise $25\geq (7y-5)(7z-5)\geq (7\cdot 2-5)^2=81$ olacağından çelişki olur. Dolayısıyla $z=1$ olmalıdır. $25\geq 2(7y-5)$ olur. $y$'yi yalnız bırakırsak $\dfrac{5}{2}\geq y$ olur ve buradan $y=1$ veya $y=2$ olabileceği görülür.

$iia)$ $y=1$ ise ana denklemde $(x+2)^2=7x$ denklemi elde edilir fakat bu denklemin tamsayı çözümü yoktur.

$iib)$ $y=2$ ise ana denklemde $(x+3)^2=14x$ denklemi elde edilir fakat bu denklemin de tamsayı çözümü yoktur. Dolayısıyla $n=7$ için denklemin çözümü yoktur.

Soruda bizden çözüm olan $n$ değerleri istendiğinden $n=1,2,3,4,5,6,8,9$ için çözüme örnek vermemiz yeterlidir. $(n,x,y,z)=(1,9,9,9)$, $(2,8,4,4)$, $(3,3,3,3)$, $(4,4,2,2)$, $(5,5,4,1)$, $(6,3,2,1)$, $(8,2,1,1)$, $(9,1,1,1)$ örnek çözümleri vardır. Dolayısıyla çözümü mümkün kılan $n$ değerleri $n=1,2,3,4,5,6,8,9$ bulunur.
Gerçek hikayeler aslında söylenmeyenlerdir.

Çevrimdışı Metin Can Aydemir

  • G.O Genel Moderator
  • Geo-Maniac
  • ********
  • İleti: 1139
  • Karma: +9/-0
Ynt: Diyafont Denklemler Çalışma Soruları ($138$ Tane)
« Yanıtla #94 : Ekim 26, 2020, 12:36:04 ös »
$88)$ $x^2+84x+2008$ ifadesini düzenlersek $$y^2=x^2+84x+2008=(x+42)^2+244\Rightarrow (y-x-42)(y+x+42)=244=2^2\cdot 61$$ olur. $(y-x-42)+(y+x+42)=2y$ olduğundan ya iki çarpan da tektir ya da çifttir. Çarpımları çift olduğundan iki terim de çift olmalıdır. Ayrıca $y+x+42\geq y-x-42$ olduğundan $y+x+42=122$ ve $y-x-42=2$ olmalıdır. Bu iki ifadeyi toplarsak $y=62$ ve bunu denkleme koyarsak $x=18$ bulunur. $\boxed{x+y=80}$ bulunur.
Gerçek hikayeler aslında söylenmeyenlerdir.

Çevrimdışı Metin Can Aydemir

  • G.O Genel Moderator
  • Geo-Maniac
  • ********
  • İleti: 1139
  • Karma: +9/-0
Ynt: Diyafont Denklemler Çalışma Soruları ($138$ Tane)
« Yanıtla #95 : Ekim 26, 2020, 12:45:06 ös »
$89)$ Öncelikle $A=0$ durumuna bakalım. Denklem $(10B+C)(B+C)=2005$ durumuna gelir. $(10B+C)(B+C)\leq (10\cdot 9+9)(9+9)=1782$ olduğundan $(10B+C)(B+C)=2005$ denkleminin çözümü yoktur.

$A,B,C$ rakam olduğundan $A\neq 0$ ise $\overline{ABC}$ bir üç basamaklı sayı belirtir. $\overline{ABC}(A+B+C)=2005=5\cdot 401$ olduğundan $\overline{ABC}$, $2005$ sayısının bir bölenidir ve tüm bölenler $1,5,401,2005$ olduğundan sadece $\overline{ABC}=401$ olabilir. Bu durumda $A+B+C=5$ sağladığından $(A,B,C)=(4,0,1)$ çözümü bulunur. $\boxed{A=4}$'dür.
Gerçek hikayeler aslında söylenmeyenlerdir.

Çevrimdışı Metin Can Aydemir

  • G.O Genel Moderator
  • Geo-Maniac
  • ********
  • İleti: 1139
  • Karma: +9/-0
Ynt: Diyafont Denklemler Çalışma Soruları ($138$ Tane)
« Yanıtla #96 : Ekim 26, 2020, 12:51:11 ös »
$90)$ $x\in\mathbb{N}$ olsun ve $x^2=71p+1$ sağlasın. $x^2-1=(x-1)(x+1)=71p$ bulunur. $x+1>x-1$ olduğundan ve $71$ de bir asal sayı olduğundan $((x-1),(x+1))=(1,71p),(71,p),(p,71)$ olabilir. $x-1=1$ ise $x+1=3=71p$ bulunur fakat çözüm yoktur.

Eğer $x-1=71$ ise $x+1=p=73$ bulunur. Gerçekten de $p=73$ için $71\cdot 73+1=72^2$ sağlar.

Eğer $x+1=71$ ise $x-1=p=69$ bulunur fakat $69$ asal sayı değidir. Dolayısıyla şartı sağlayan tek asal sayı $\boxed{p=73}$'dür.
Gerçek hikayeler aslında söylenmeyenlerdir.

Çevrimdışı Metin Can Aydemir

  • G.O Genel Moderator
  • Geo-Maniac
  • ********
  • İleti: 1139
  • Karma: +9/-0
Ynt: Diyafont Denklemler Çalışma Soruları ($138$ Tane)
« Yanıtla #97 : Ekim 26, 2020, 06:03:35 ös »
$91)$ Verilen eşitlikte sağ taraf $7$ ile bölündüğünden sol taraf da bölünmelidir. Dolayısıyla $p,q,r$'den en az biri $7$ olmalıdır. Genelliği bozmadan $r=7$ olsun. Denklemi düzenlersek, $$pq=p+q+7\Rightarrow pq-p-q+1=8\Rightarrow (p-1)(q-1)=8$$ bulunur. Bu eşitliği $8$'in çarpanlarına ayırarak çözebiliriz. Genelliği bozmadan $p\geq q$ olsun dersek $(p,q)=(9,2),(5,3)$ bulunur. $9$ asal sayı olmadığından $(p,q)=(5,3)$ olmalıdır. Buradan $\boxed{p+q+r=3+5+7=15}$ bulunur.
Gerçek hikayeler aslında söylenmeyenlerdir.

Çevrimdışı Metin Can Aydemir

  • G.O Genel Moderator
  • Geo-Maniac
  • ********
  • İleti: 1139
  • Karma: +9/-0
Ynt: Diyafont Denklemler Çalışma Soruları ($138$ Tane)
« Yanıtla #98 : Ekim 26, 2020, 06:16:17 ös »
$92)$ $n-76=a^3$ ve $n+76=b^3$ yazalım. $b^3-a^3=152$ bulunur. $$b^3-a^3=(b-a)(a^2+b^2+ab)=152=2^3\cdot 19$$ bulunur. $b^3-a^3=152$ olduğundan $a$ ve $b$'nin pariteleri aynı olmalıdır. Dolayısıyla $b-a$ çift olacaktır. Ayrıca $a^2+b^2+ab=(b-a)^2+3ab$ olduğundan $a^2+b^2+ab>(b-a)^2$ olacaktır. Bu şartlar altında olası durumlar şunlardır; $(b-a, a^2+b^2+ab)=(2,76),(4,38)$ bulunur.

$i)$ $b-a=2$ ve $a^2+b^2+ab=76$ ise $b=a+2$ yazarsak $$a^2+(a+2)^2+a(a+2)=76\Rightarrow a^2+2a-24=(a+6)(a-4)=0$$ bulunur. $a$ pozitif olduğundan $a=4$ ve $b=a+2=6$ olacaktır. Bu değerler için $n=140$ bulunur.

$ii)$ $b-a=4$ ve $a^2+b^2+ab=38$ ise $b=a+4$ için $$a^2+(a+4)^2+a(a+4)=38\Rightarrow 3a^2+12a-22=0$$ bulunur fakat buradan tamsayı çözümü gelmez.

Şartı sağlayan tek sayı $\boxed{n=140}$'dır.
Gerçek hikayeler aslında söylenmeyenlerdir.

Çevrimdışı Metin Can Aydemir

  • G.O Genel Moderator
  • Geo-Maniac
  • ********
  • İleti: 1139
  • Karma: +9/-0
Ynt: Diyafont Denklemler Çalışma Soruları ($138$ Tane)
« Yanıtla #99 : Ekim 26, 2020, 11:52:57 ös »
$93)$ Herhangi bir $a>0$ ve $a\neq 1$ reel sayısı için $\log_{a}{x}=0$ ise $x=1$'dir ve $\log_{a}{y}=1$ ise $y=a$'dır. Dolayısıyla $$\log_{2}{\left (\log_{2^a}{\left (\log_{2^b}{2^{1000}}\right )}\right )}=0\Rightarrow \log_{2^a}{\left (\log_{2^b}{2^{1000}}\right )}=1\Rightarrow \log_{2^b}{2^{1000}}=2^a$$ bulunur. $\log_{a^x}{b^y}=\dfrac{y}{x}\log_{a}{b}$ olduğundan $\log_{2^b}{2^{1000}}=\dfrac{1000}{b}\log_{2}{2}=\dfrac{1000}{b}=2^a$ bulunur. $1000=2^3\cdot 5^3$ olduğundan ve $2^a$ sayısının da $1000$'in bir böleni olmasından dolayı $a=1,2,3$ olabilir. Bu değerler için $b$ değerlerini de hesaplayabiliriz.

Tüm çözümler $(a,b)=(1,500),(2,250),(3,125)$'dir. Bu çözümler için $a+b$ ifadesi $501,252,128$ değerlerini alır. Bu değerlerin toplamı $501+252+128=\boxed{881}$ bulunur.
« Son Düzenleme: Kasım 04, 2020, 06:37:07 ös Gönderen: metonster »
Gerçek hikayeler aslında söylenmeyenlerdir.

Çevrimdışı Metin Can Aydemir

  • G.O Genel Moderator
  • Geo-Maniac
  • ********
  • İleti: 1139
  • Karma: +9/-0
Ynt: Diyafont Denklemler Çalışma Soruları ($138$ Tane)
« Yanıtla #100 : Ekim 27, 2020, 12:06:01 öö »
$94)$ Öncelikle şu görülebilir ki $5>y$ ise denklemin sol tarafı negatif olmaktadır. Dolayısıyla negatif $y$ değerleri için çözüm yoktur. Ayrıca $(x,y)$ bir çözümse $(-x,y)$ de bir çözüm olacağından genelliği bozmadan $x$'i pozitif alabiliriz. ($x=0$ için denklemin çözümü olmadığı barizdir.) $$4^y-615=x^2\Rightarrow 4^y-x^2=(2^y-x)(2^y+x)=615=3\cdot 5\cdot 41$$ elde edilir. $2^y-x+2^y+x=2^{y+1}$ olduğundan $615$'in çarpanları arasından toplamı $2$'nin kuvveti olan ve çarpımları $615$ olan ikilileri bulmalıyız. Çarpımları $615$ olan tüm ikililer $(1,615)$, $(3,205)$, $(5,123)$, $(15,41)$'dir. Bunlardan toplamı $2$'nin kuvveti olan sadece $(5,123)$ ikilisidir. Dolayısıyla $2^y-x=5$ ve $2^y+x=123$ olmalıdır, bu iki denklemden $(x,y)=(59,6)$ bulunur. Başta da belirttiğimiz gibi $(x,y)=(59,6)$ çözümse $(x,y)=(-59,6)$ da bir çözümdür. Tüm çözümler, $\boxed{(x,y)=(59,6),(-59,6)}$ bulunur.
Gerçek hikayeler aslında söylenmeyenlerdir.

Çevrimdışı Metin Can Aydemir

  • G.O Genel Moderator
  • Geo-Maniac
  • ********
  • İleti: 1139
  • Karma: +9/-0
Ynt: Diyafont Denklemler Çalışma Soruları ($138$ Tane)
« Yanıtla #101 : Ekim 28, 2020, 11:25:21 ös »
$95)$ Verilen ifadeyi düzenleyelim, $$2(x^2+y^2)+x+y=5xy\Rightarrow 2(x+y)^2+x+y=9xy$$ $x+y=a$ ve $xy=b$ için $2a^2+a=9b$ bulunur. Ayrıca, $$(x+y)^2-4xy=(x-y)^2\Rightarrow a^2-4b=(x-y)^2\Rightarrow \sqrt{a^2-4b}\in \mathbb{Z}$$ $$\sqrt{a^2-4b}\in \mathbb{Z}\Rightarrow \sqrt{9a^2-36b}\in \mathbb{Z}\Rightarrow \sqrt{9a^2-4(2a^2+a)}\in \mathbb{Z}\Rightarrow \sqrt{a^2-4a}\in \mathbb{Z}$$ bulunur. Öyleyse $a^2-4a=t^2$ olacak şekilde bir $t$ tamsayısı vardır. $$a^2-4a+4=t^2+4\Rightarrow (a-2)^2-t^2=4\Rightarrow (a-2-t)(a-2+t)=4$$ bulunur. $(a-2-t)+(a-2+t)=2a-4$ olduğundan $(a-2-t)$ ve $(a-2+t)$ ifadeleri aynı paritedelerdir ve ayrıca çarpımları çift olduğundan ikisi de çift olmalıdır. Dolayısıyla ihtimaller, $(a-2-t,a-2+t)=(-2,-2)$ ve $(a-2-t,a-2+t)=(2,2)$'dir. Buradan $a=0$ ve $a=4$ çözümleri bulunur. Bu değerler için $2a^2+a=9b$ olduğundan $b$ değerlerini de bulabiliriz. $(a,b)=(0,0)$ ve $(a,b)=(4,4)$ bulunur. $x+y=a$ ve $xy=b$ olduğundan $\boxed{(x,y)=(0,0),(2,2)}$ bulunur. Tüm çözümler bunlardır.
Gerçek hikayeler aslında söylenmeyenlerdir.

Çevrimdışı Metin Can Aydemir

  • G.O Genel Moderator
  • Geo-Maniac
  • ********
  • İleti: 1139
  • Karma: +9/-0
Ynt: Diyafont Denklemler Çalışma Soruları ($138$ Tane)
« Yanıtla #102 : Mayıs 08, 2022, 01:34:54 öö »
$41)$ Öncelikle $p=q$ için çözüm var mı araştıralım. Denklem $p^{m+n}=4p^2+1$ olur. Sol taraf $p$ ile bölündüğünden $4p^2+1$ ve $1$ sayısı da $p$ ile bölünür. Yani $p=1$ olmalıdır fakat denklemi sağlamaz. Şimdi genelliği bozmadan $p>q$ kabul edelim.

Eğer $m\geq 3$ ise $p^mq^n\geq p^3$ ve $(p+q)^2+1\leq (p+p)^2+1=4p^2+1$ olduğundan $$p^3\leq 4p^2+1\Rightarrow 4\geq p$$ olacaktır. $p=1$ olamaz çünkü $p>q\geq 1$'dir.

$p=2$ ise $q=1$ olmalıdır fakat $2^m=(2+1)^2+1$ denkleminin çözümü yoktur.

$p=3$ ise $q=1$ veya $q=2$ olmalıdır. İkisi için de denklemin çözümü yoktur.

$p=4$ ise $q=1$ veya $q=2$ veya $q=3$ olmalıdır. Üçü için de denklemin çözümü yoktur.

i) $m=2$ ise $q=1$ için denklem sağlanmaz çünkü $p^2=(p+1)^2+1$ denkleminin çözümü yoktur. Yani $p>q\geq 2$ diyebiliriz. Denklemi açıp düzenlersek $p^2(q^n-1)-2pq-(q^2+1)=0$ olur. Bu ikinci dereceden bir denklemdir ve çözümünü yapabiliriz. Denklemin köklerinin çarpımı, Vieta teoreminden, $-\dfrac{q^2+1}{q^n-1}$ olduğundan köklerden biri pozitif, diğeri negatiftir. Bu yüzden $$p=\dfrac{q+\sqrt{q^{n+2}-q^n+1}}{q^n-1}$$ olacaktır. İfade $1$'den büyük olduğundan $$q+\sqrt{q^{n+2}-q^n+1}>q^n-1\implies q^{n+2}-q^n+1\geq (q^n-q)^2\implies q^{n+2}+2q^{n+1}+1\geq q^{2n}+q^n+q^2$$ $$3q^{n+2}+q^2\geq q^{n+2}+2q^{n+1}+1\geq q^{2n}+q^n+q^2\implies 3q^{n+2}\geq q^{2n}+q^n\implies 3q^2\geq q^n+1$$ Buradan $n\geq 4$ için çelişki gelir. $n=3$ için ise $q\leq 3$ olmalıdır aksi taktirde $3q^2< q^3+1$ olur. $n=3$ için de $q=2$ ve $q=3$'ten çözüm gelmez.

$n=2$ ise $(pq)^2=(p+q)^2+1$ olur fakat aralarında $1$ fark olan tek olası tamkareler $1$ ve $0$'dır. Bu bir çelişkidir.

$n=1$ ise denklem $p^2q=(p+q)^2+1$ şekline gelir. Bu denklemi de düzenlersek $q^2+q(2p-p^2)+p^2+1=0$ olur. $$x^2+x(2p-p^2)+p^2+1=0$$ denkleminin kökleri $q_1>q_2$ olsun (eşitlik durumunda $q^2=p^2+1$ olur). Vieta teoreminden $$q_1+q_2=p^2-2p$$ $$q_1q_2=p^2+1$$ Yani $p^2q=(p+q)^2+1$ denkleminin istediğimiz koşullarda çözümü varsa hem $q_1$ hem de $q_2$ pozitif tamsayıdır. En az biri $p$'den küçük olduğundan ve çarpımları $p^2$'den büyük olduğundan $q_1>p>q_2$ olacaktır ($q_1,q_2\neq p$ olmalıdır çünkü $q_1q_2=p^2+1$ sayısı $p$ ile bölünmez). Burada $(p,q_2)$ denklemimizin aradığımız çözümüdür. Vieta'dan elde ettiğimiz eşitlikleri birbirinden çıkartırsak $$(q_1-1)(q_2-1)=2p+2\geq (p-1)(q_2-1)$$ elde edilir. $q_2\geq 4$ ise $2p+2\geq 3p-3$ elde edilir yani $5\geq p$ olur. Tek olası durum $(p,q_2)=(5,4)$'dür ama denkleme çözüm değildir. O halde $q_2\leq 3$ olmalıdır. Buradan $q_2=2$ için $p=5$ çözümü gelir.

ii) Eğer $m=1$ ise $q$ çift olamaz çünkü aksi taktirde $(p+q)$ tek olacaktır ama $(p+q)^2+1\equiv 2\pmod{4}$ olduğundan $n=1$ olacaktır. Buradan da çözüm gelmez. Dolayısıyla $n\geq 2$ ve $q\geq 3$ diyebiliriz. Denklemi düzenlersek $$p^2-p(q^n-2q)+q^2+1=0$$ olur. $x^2-x(q^n-2q)+q^2+1=0$ denkleminin kökleri  $p_1>p_2$ olsun ($p_1=p_2$ durumunda $p_1^2=q^2+1$ olacaktır). Denklemin aradığımız şartlarda bir çözümü varsa $p_1>q>p_2\geq 2$ olmalıdır çünkü $p_1>p_2>q$ durumunda $p_1p_2>q^2+1$ olacaktır ($p_1=q$ veya $p_2=q$ veya $p_2=1$ durumunda $q$ bulunamadığı görülebilir). Buradan $$p_1+q>p_1+p_2=q^n-2q\implies p_1>q^n-3q\implies q^2+1=p_1p_2>p_1>q^n-3q$$ $n>2$ için son eşitsizlik sağlanmaz. O halde $n=2$ olmalıdır. Bu durumda Vieta teoreminden $(p_1-1)(p_2-1)=2q+2$ elde edilir. $p_2=2$ ise $p_1=2q+3$ olur ve denklemde yerine yazarsak $(p,q)=(13,5)$ çözümü gelir. $p_2>2$ ise $$ 2q+2\geq2(p_1-1)\implies q+2\geq p_1$$ elde edilir. Yani $p_1=q+1$ veya $q+2$ olacaktır. İki durumda da denklemde yerine yazınca çözüm gelmez. O halde buradan gelen tek çözüm $(p,q)=(13,5)$'dir.

Tüm çözümler, $(p,q,m,n)=(13,5,1,2),(5,13,2,1),(5,2,2,1),(2,5,1,2)$'dir.
« Son Düzenleme: Nisan 03, 2024, 06:53:51 ös Gönderen: Metin Can Aydemir »
Gerçek hikayeler aslında söylenmeyenlerdir.

Çevrimdışı Metin Can Aydemir

  • G.O Genel Moderator
  • Geo-Maniac
  • ********
  • İleti: 1139
  • Karma: +9/-0
Ynt: Diyafont Denklemler Çalışma Soruları ($138$ Tane)
« Yanıtla #103 : Haziran 09, 2022, 08:37:43 ös »
96) $x^3=1$ denkleminin karmaşık sayılarda üç tane çözümü vardır. Bunlar, $\omega=e^{\frac{2\pi i}{3}}$ olmak üzere $1$, $\omega$ ve $\omega^2$'dir ve $\omega^2+\omega+1=0$ sağlanır. Biz $x^3+y^3=z^3$ denkleminin tamsayılardan daha büyük bir küme olan $\mathbb{Z}[\omega]=\{a+b\omega\mid a,b\in \mathbb{Z}\}$'de çözümümlerine bakacağız. Buradaki gönderide $\mathbb{Z}[\omega]$'nin birim veya $0$ olmayan her elemanının tek şekilde çarpanlarına ayrılabildiğini göstermiştik ($\mathbb{Z}[\omega]$'de birim elemanlar $1,-1,\omega,-\omega,1+\omega,-1-\omega$'dır).

$x^3+y^3=z^3$ denkleminde eğer $\text{EBOB}(x,y,z)=d$ ise herhangi bir $(x,y,z)$ çözümü için $\left(\dfrac{x}{d},\dfrac{y}{d},\dfrac{z}{d}\right)$ de çözüm olacağından genelliği bozmadan $\text{EBOB}(x,y,z)=1$ kabul edebiliriz. Tüm çözümleri bulmak için elde edeceğimiz ilkel çözümleri $d$ ile çarpmamız yeterli olacaktır. Bu denklem için $x,y,z$'nin aralarında asal olması ikişerli olarak da aralarında asal olması demektir. Şimdi $x$ ve $y$'nin $\mathbb{Z}[\omega]$'de aralarında asal olup olmadığını kontrol edelim. Şimdilik $xyz\neq 0$ olarak kabul edelim.

$\alpha\in\mathbb{Z}[\omega]$ olmak üzere $\alpha\mid x$ ve $\alpha \mid y$ olsun. $\alpha=a+b\omega$ ve $\text{EBOB}(a,b)=d$ olsun. $\alpha\beta=x$ ve $\alpha=d\theta$ olacak şekilde bir $\beta,\theta\in \mathbb{Z}[\omega]$ vardır. Dolayısıyla $d\cdot (\beta\theta)=x$ olur. $\beta\theta\in\mathbb{Z}[\omega]$ ve $d,x\in\mathbb{Z}$ olduğundan $\beta\theta\in\mathbb{Z}$ olmalıdır ve buradan tamsayılar için $d\mid x$ bulunur. Benzer şekilde $y$ için de aynısını yapabiliriz. $x$ ve $y$ aralarında asal olduğundan $d=1$ ve $a$ ve $b$ aralarında asal olmalıdır. $\alpha \neq 0$ olduğundan $\text{EBOB}(a,b)=1$ olmasında istisna olabilecek durumlar $(a,b)=(0,\pm 1)$ ve $(a,b)=(\pm 1,0)$'dır. İki durumda da $\alpha$ birim çıkar. O yüzden $a,b\neq 0$ diyebiliriz. $\beta=c+d\omega$ dersek $$\alpha\beta=(a+b\omega)(c+d\omega)=(ac-bd)+(ad+bc-bd)\omega\in \mathbb{Z}\implies ad+bc=bd$$ $a$ ve $b$'yi aralarında asal bulduğumuzdan son eşitlikten $b\mid d$ elde edilir. $t\in \mathbb{Z}$ için $d=bt$ yazıp $b$'leri sadeleştirirsek $at+c=bt$ olur. $c=(b-a)t$ ve $d=bt$ olduğundan $$\alpha\beta=ac-bd=t(ab-a^2-b^2)=x$$ elde edilir. $a^2+b^2-ab\geq 1$ olduğunu yukarıda bağlantısını bıraktığım gönderide 3. teoremin ispatından görebilirsiniz. $a^2+b^2-ab$ ifadesi hem $x$ hem de $y$'yi böldüğünden $1$'e eşit olmalıdır. $$1=a^2+b^2-ab\geq a^2+b^2-|ab|\geq |ab|\geq 0$$ olmasını kullanarak bu denklemi çözebiliriz. Her durumda $\alpha$ birim olarak bulunur. Yani $x$ ve $y$ için $\mathbb{Z}[\omega]$'da da aralarında asal diyebiliriz.

Ana denkleme dönersek bu denklemi $\mathbb{Z}[\omega]$'da çarpanlarında ayırırsak $$x^3+y^3=(x+y)(x+\omega y)(x+\omega^2 y)=(x+y)(x+\omega y)(x-y-\omega y)=z^3$$ olarak buluruz. Şimdi $(x+y)$, $(x+\omega y)$ ve $(x-y-\omega y)$ ifadelerinin ortak bölenlerini inceleyelim. Aralarında asal olmaları durumunu kenarda tutalım ve farz edelim ki yukarıdakinden farklı bir sıfırdan farklı ve birim olmayan $\alpha$ hem $x+y$ hem de $x+\omega y$'yi bölüyor. Dolayısıyla ikisinin farklı olan $(\omega -1)y$'yi de bölecektir. $\omega -1$ indirgenemezdir (aksini kabul edip kolayca çelişki bulabilirsiniz veya norm kavramını bilen kişiler normunun $3$ olmasından dolayı bunu kolaylıkla görecektir). Eğer $\alpha$ ile $y$ aralarında asal değilse herhangi bir ortak bölenleri $x$ de böleceğinden çelişki çıkar. Dolayısıyla $\alpha=\omega-1$ olmalıdır. Eğer $\omega-1\mid x+y$ ise diğer çarpanları da böldüğünü görebiliriz. O halde elimizde iki ihtimal vardır. $\mathbb{Z}[\omega]$ bir UFD olduğundan ikişerli olarak aralarında asal $u_1,u_2,u_3\in\mathbb{Z}[\omega]$ için

$(x+y,x+\omega y, x-y-\omega y)=(u_1^3,u_2^3,u_3^3)$ veya $(x+y,x+\omega y, x-y-\omega y)=((\omega-1)u_1^3,(\omega-1)u_2^3,(\omega-1)u_3^3)$ olacaktır. İkinci durumu önce inceleyelim. $u_1=a+b\omega$ dersek $(\omega-1)(a+b\omega)^3$ tamsayı olması gerektiğinden $$(\omega-1)((a^3-3ab^2+b^3)+(3a^2b-3ab^2)\omega)=(-a^3-3a^2b+6ab^2-b^3)+(a^3-6a^2b+3ab^2+b^3)\omega\in\mathbb{Z}\implies a^3-6a^2b+3ab^2+b^3=0$$ Eğer $b=0$ ise $a=0$ bulunur. Bu durumda da $z=0$ olur. Çelişki. $b\neq 0$ için her tarafı $b^3$'e bölüp $\dfrac{a}{b}=t$ dersek $$t^3-6t^2+3t+1=0$$ elde edilir ama bu denklemin rasyonel kökü yoktur. Buradan çözüm gelmez.

Eğer $(x+y,x+\omega y,x-y-\omega y)=(u_1^3,u_2^3,u_3^3)$ ise $u_2=c+d\omega$ diyelim. Buradan $$u_2^3=x+y\omega=(c^3-3cd^2+d^3)+(3c^2d-3cd^2)\omega\implies y=3c^2d-3cd^2$$ elde edilir. Yani $3\mid y$'dir. Eğer $x^3+y^3$ ifadesini $(x+y)(y+\omega x)(y-x-\omega x)$ olarak çarpanlarına ayırsaydık, $x$ de $3$'ün katı bulunacaktı. Bu $x$ ve $y$'nin aralarında asal olmasıyla çelişir.

Yani $xyz=0$ olmalıdır. İfadede sırayla $x=0$, $y=0$ ve $z=0$'ı denersek $k\in \mathbb{Z}$ için $(x,y,z)=(0,k,k), (k,0,k),(k,-k,0)$ çözümleri elde edilir. Tüm çözümler bunlardır.
« Son Düzenleme: Haziran 09, 2022, 10:42:46 ös Gönderen: Metin Can Aydemir »
Gerçek hikayeler aslında söylenmeyenlerdir.

Çevrimdışı Metin Can Aydemir

  • G.O Genel Moderator
  • Geo-Maniac
  • ********
  • İleti: 1139
  • Karma: +9/-0
Ynt: Diyafont Denklemler Çalışma Soruları ($138$ Tane)
« Yanıtla #104 : Haziran 09, 2022, 09:18:20 ös »
97) Verilen eşitliği bir polinoma çevirelim. $$P(t)=t^{x_{2011}}+t^{x_{2010}}+\cdots+t^{x_1}-t^{x_0}$$ polinomunun pozitif tamsayı olan bir kökü olmasını istiyoruz. $P(1)=2010$ ve $P(m)=0$ olduğundan ve tüm katsayılar tamsayı olduğundan $$m-1\mid 2010$$ olacaktır. $2010=2\cdot 3\cdot 5\cdot 67$ olduğundan olası $16$ adet $m$ pozitif tamsayısı vardır. Bu değerlerin hepsi için şarta uygun bir $x_i$'ler elde edebileceğimizi göstersek yeterlidir. Öncelikle $$\underbrace{m^i+m^i+\cdots+m^i}_{m\text{  adet}}=m^{i+1}$$ olduğunu gözlemleyelim. $(m-1)k=2010$ dersek $$\underbrace{m^0+m^0+\cdots+m^0}_{m\text{  adet}}=m^1$$ $$\underbrace{m^1+m^1+\cdots+m^1}_{m\text{  adet}}=m^{2}$$ $$\vdots$$ $$\underbrace{m^{k-1}+m^{k-1}+\cdots+m^{k-1}}_{m\text{  adet}}=m^{k}$$ eşitliklerini toplarsak $$\underbrace{m^0+m^0+\cdots+m^0}_{m\text{  adet}}+\underbrace{m^1+m^1+\cdots+m^1}_{(m-1)\text{  adet}}+\cdots+\underbrace{m^{k-1}+m^{k-1}+\cdots+m^{k-1}}_{(m-1)\text{  adet}}=m^{k}$$ elde edilir. Son eşitliğin sol tarafında $m+(m-1)(k-1)=(m-1)k+1=2011$ adet $m$'nin kuvveti vardır. Dolayısıyla yukarıda bulduğumuz $16$ adet sayının hepsi için istenilen $x_i$ negatif olmayan tamsayılarını bulabiliriz. Cevap $\boxed{16}$'dır.
Gerçek hikayeler aslında söylenmeyenlerdir.

 


Sitemap 1 2 3 4 5 6 7 8 9 10 11 12 13 14 15 16 17 18 19 20 21 22 23 24 25 26 27 28 29 30 31 32 33 34 35 36 37 
SimplePortal 2.3.3 © 2008-2010, SimplePortal